Proof of convergence for non-absolute sequenceProving the convergence of $a_n = frac{n}{n+sqrt n}$Convergence...

How to make healing in an exploration game interesting

Professor being mistaken for a grad student

How difficult is it to simply disable/disengage the MCAS on Boeing 737 Max 8 & 9 Aircraft?

Are there verbs that are neither telic, or atelic?

If I can solve Sudoku can I solve Travelling Salesman Problem(TSP)? If yes, how?

Python if-else code style for reduced code for rounding floats

Instead of Universal Basic Income, why not Universal Basic NEEDS?

Does Mathematica reuse previous computations?

How can I track script which gives me "command not found" right after the login?

How to create the Curved texte?

Who is flying the vertibirds?

Interplanetary conflict, some disease destroys the ability to understand or appreciate music

Do I need to be arrogant to get ahead?

Are ETF trackers fundamentally better than individual stocks?

Does someone need to be connected to my network to sniff HTTP requests?

If curse and magic is two sides of the same coin, why the former is forbidden?

How Could an Airship Be Repaired Mid-Flight

Unexpected result from ArcLength

Why is the President allowed to veto a cancellation of emergency powers?

Dice rolling probability game

Credit cards used everywhere in Singapore or Malaysia?

Official degrees of earth’s rotation per day

How to explain that I do not want to visit a country due to personal safety concern?

Time travel from stationary position?



Proof of convergence for non-absolute sequence


Proving the convergence of $a_n = frac{n}{n+sqrt n}$Convergence of Monotone Sequence in Affine-Extended RealsNon-increasing Monotone Sequence Convergence ProofEvaluating $lim_{n to infty} sumlimits_{k=1}^n frac{1}{k 2^k} $Suppose that the function $f:Irightarrowmathbb{R}$ is monotonically increasing and bounded. Prove that the $lim_{xrightarrow a}f(x)$ existsCheck my Work: Series ConvergenceProving that the limit of the product of these two sequences is zeroA sufficient condition for a sequence to converge if arithmetic mean of the sequence converges?Convergence of sequence of real functionIf a sequence of absolute values is bounded, does it then converge?













0












$begingroup$


I've established that the bounded sequence $A_{n}=sumlimits_{k=1}^{n}|x_{k}|$ converges, since it is bounded and monotonically increasing.
Now I'm trying to prove that the sequence $B_{n}=sumlimits_{k=1}^{n}x_{k}$ (from the same $x_{k}$) is also convergent. I'm trying to use the following fact: $|B_{n}-B_{m}|=A_{n}-A_{m}$ for $n>m$. Now since $A_{n}$ is convergent I know there exists a $N in mathbb{N}$ such that when $n,m > mathbb{N}$ and $epsilon>0$ we have $|A_{n}-L|<epsilon$ and $|A_{m}-L|<epsilon$.



Can I then just say $|B_{n}-B_{m}|=A_{n}-A_{m} = A_{n}-L+L-A_{m} leq |A_{n}-L|+|A_{m}-L| leq 2epsilon$?



I'm a bit confused since I'm not quite sure if it is actually true that $A_{n}$ converges to $L$ in stead of $|x_{k}|$. Because I think that if $A_{n}$ converges to $L$, then I would suppose $x_{k}$ would have to tend to $0$, but I don't see why this would be the case.



As you are probably able to see, I'm confused and any suggestions would be more than welcome.










share|cite|improve this question









$endgroup$

















    0












    $begingroup$


    I've established that the bounded sequence $A_{n}=sumlimits_{k=1}^{n}|x_{k}|$ converges, since it is bounded and monotonically increasing.
    Now I'm trying to prove that the sequence $B_{n}=sumlimits_{k=1}^{n}x_{k}$ (from the same $x_{k}$) is also convergent. I'm trying to use the following fact: $|B_{n}-B_{m}|=A_{n}-A_{m}$ for $n>m$. Now since $A_{n}$ is convergent I know there exists a $N in mathbb{N}$ such that when $n,m > mathbb{N}$ and $epsilon>0$ we have $|A_{n}-L|<epsilon$ and $|A_{m}-L|<epsilon$.



    Can I then just say $|B_{n}-B_{m}|=A_{n}-A_{m} = A_{n}-L+L-A_{m} leq |A_{n}-L|+|A_{m}-L| leq 2epsilon$?



    I'm a bit confused since I'm not quite sure if it is actually true that $A_{n}$ converges to $L$ in stead of $|x_{k}|$. Because I think that if $A_{n}$ converges to $L$, then I would suppose $x_{k}$ would have to tend to $0$, but I don't see why this would be the case.



    As you are probably able to see, I'm confused and any suggestions would be more than welcome.










    share|cite|improve this question









    $endgroup$















      0












      0








      0





      $begingroup$


      I've established that the bounded sequence $A_{n}=sumlimits_{k=1}^{n}|x_{k}|$ converges, since it is bounded and monotonically increasing.
      Now I'm trying to prove that the sequence $B_{n}=sumlimits_{k=1}^{n}x_{k}$ (from the same $x_{k}$) is also convergent. I'm trying to use the following fact: $|B_{n}-B_{m}|=A_{n}-A_{m}$ for $n>m$. Now since $A_{n}$ is convergent I know there exists a $N in mathbb{N}$ such that when $n,m > mathbb{N}$ and $epsilon>0$ we have $|A_{n}-L|<epsilon$ and $|A_{m}-L|<epsilon$.



      Can I then just say $|B_{n}-B_{m}|=A_{n}-A_{m} = A_{n}-L+L-A_{m} leq |A_{n}-L|+|A_{m}-L| leq 2epsilon$?



      I'm a bit confused since I'm not quite sure if it is actually true that $A_{n}$ converges to $L$ in stead of $|x_{k}|$. Because I think that if $A_{n}$ converges to $L$, then I would suppose $x_{k}$ would have to tend to $0$, but I don't see why this would be the case.



      As you are probably able to see, I'm confused and any suggestions would be more than welcome.










      share|cite|improve this question









      $endgroup$




      I've established that the bounded sequence $A_{n}=sumlimits_{k=1}^{n}|x_{k}|$ converges, since it is bounded and monotonically increasing.
      Now I'm trying to prove that the sequence $B_{n}=sumlimits_{k=1}^{n}x_{k}$ (from the same $x_{k}$) is also convergent. I'm trying to use the following fact: $|B_{n}-B_{m}|=A_{n}-A_{m}$ for $n>m$. Now since $A_{n}$ is convergent I know there exists a $N in mathbb{N}$ such that when $n,m > mathbb{N}$ and $epsilon>0$ we have $|A_{n}-L|<epsilon$ and $|A_{m}-L|<epsilon$.



      Can I then just say $|B_{n}-B_{m}|=A_{n}-A_{m} = A_{n}-L+L-A_{m} leq |A_{n}-L|+|A_{m}-L| leq 2epsilon$?



      I'm a bit confused since I'm not quite sure if it is actually true that $A_{n}$ converges to $L$ in stead of $|x_{k}|$. Because I think that if $A_{n}$ converges to $L$, then I would suppose $x_{k}$ would have to tend to $0$, but I don't see why this would be the case.



      As you are probably able to see, I'm confused and any suggestions would be more than welcome.







      real-analysis sequences-and-series limits analysis






      share|cite|improve this question













      share|cite|improve this question











      share|cite|improve this question




      share|cite|improve this question










      asked Mar 10 at 22:10









      MathbeginnerMathbeginner

      1738




      1738






















          1 Answer
          1






          active

          oldest

          votes


















          1












          $begingroup$

          You should use Cauchy criterion.



          For $varepsilon > 0$, because $(A_n)$ converges there exists $N$ such that for $p > q geq N$,
          $$|A_p - A_q| leq varepsilon, text{ i.e. } sum_{k=q+1}^p |x_k| leq varepsilon$$



          So for $p>q geq N$,
          $$|B_p - B_q| = left| sum_{k=q+1}^p x_k right| leq sum_{k=q+1}^p |x_k| leq varepsilon$$



          This proves that $(B_n)$ is a Cauchy sequence, and if your space is complete, it converges.






          share|cite|improve this answer











          $endgroup$













          • $begingroup$
            I see what you mean and how this would work. But does that also mean my approach is wrong?
            $endgroup$
            – Mathbeginner
            Mar 10 at 22:16










          • $begingroup$
            You say that $|B_n - B_m|=A_n-A_m$, but this has no reason to be true in general...
            $endgroup$
            – TheSilverDoe
            Mar 10 at 22:19










          • $begingroup$
            I would argue that $|B_{n}-B_{m}|=|sumlimits_{k=1}^{n} x_{k}-sumlimits_{k=1}^{m} x_{k}|=|sumlimits_{k=m+1}^{n} x_{k}|=sumlimits_{k=m+1}^{n} |x_{k}|=A_{n}-A_{m}$. Right?
            $endgroup$
            – Mathbeginner
            Mar 10 at 22:22












          • $begingroup$
            $|sum x_k|$ is not the same thing as $sum |x_k|$. The basic fact about $|.|$ is the triangle inequality $|sum x_k| leq sum |x_k|$. If you never heard about that, it is normal you are confused with absolute convergence questions ;)
            $endgroup$
            – TheSilverDoe
            Mar 10 at 22:24










          • $begingroup$
            You're right, I should have written $|B_{n}-B_{m}| leq A_{n}-A_{m}$, but then I could use the same road I did, right?
            $endgroup$
            – Mathbeginner
            Mar 10 at 22:26











          Your Answer





          StackExchange.ifUsing("editor", function () {
          return StackExchange.using("mathjaxEditing", function () {
          StackExchange.MarkdownEditor.creationCallbacks.add(function (editor, postfix) {
          StackExchange.mathjaxEditing.prepareWmdForMathJax(editor, postfix, [["$", "$"], ["\\(","\\)"]]);
          });
          });
          }, "mathjax-editing");

          StackExchange.ready(function() {
          var channelOptions = {
          tags: "".split(" "),
          id: "69"
          };
          initTagRenderer("".split(" "), "".split(" "), channelOptions);

          StackExchange.using("externalEditor", function() {
          // Have to fire editor after snippets, if snippets enabled
          if (StackExchange.settings.snippets.snippetsEnabled) {
          StackExchange.using("snippets", function() {
          createEditor();
          });
          }
          else {
          createEditor();
          }
          });

          function createEditor() {
          StackExchange.prepareEditor({
          heartbeatType: 'answer',
          autoActivateHeartbeat: false,
          convertImagesToLinks: true,
          noModals: true,
          showLowRepImageUploadWarning: true,
          reputationToPostImages: 10,
          bindNavPrevention: true,
          postfix: "",
          imageUploader: {
          brandingHtml: "Powered by u003ca class="icon-imgur-white" href="https://imgur.com/"u003eu003c/au003e",
          contentPolicyHtml: "User contributions licensed under u003ca href="https://creativecommons.org/licenses/by-sa/3.0/"u003ecc by-sa 3.0 with attribution requiredu003c/au003e u003ca href="https://stackoverflow.com/legal/content-policy"u003e(content policy)u003c/au003e",
          allowUrls: true
          },
          noCode: true, onDemand: true,
          discardSelector: ".discard-answer"
          ,immediatelyShowMarkdownHelp:true
          });


          }
          });














          draft saved

          draft discarded


















          StackExchange.ready(
          function () {
          StackExchange.openid.initPostLogin('.new-post-login', 'https%3a%2f%2fmath.stackexchange.com%2fquestions%2f3142980%2fproof-of-convergence-for-non-absolute-sequence%23new-answer', 'question_page');
          }
          );

          Post as a guest















          Required, but never shown

























          1 Answer
          1






          active

          oldest

          votes








          1 Answer
          1






          active

          oldest

          votes









          active

          oldest

          votes






          active

          oldest

          votes









          1












          $begingroup$

          You should use Cauchy criterion.



          For $varepsilon > 0$, because $(A_n)$ converges there exists $N$ such that for $p > q geq N$,
          $$|A_p - A_q| leq varepsilon, text{ i.e. } sum_{k=q+1}^p |x_k| leq varepsilon$$



          So for $p>q geq N$,
          $$|B_p - B_q| = left| sum_{k=q+1}^p x_k right| leq sum_{k=q+1}^p |x_k| leq varepsilon$$



          This proves that $(B_n)$ is a Cauchy sequence, and if your space is complete, it converges.






          share|cite|improve this answer











          $endgroup$













          • $begingroup$
            I see what you mean and how this would work. But does that also mean my approach is wrong?
            $endgroup$
            – Mathbeginner
            Mar 10 at 22:16










          • $begingroup$
            You say that $|B_n - B_m|=A_n-A_m$, but this has no reason to be true in general...
            $endgroup$
            – TheSilverDoe
            Mar 10 at 22:19










          • $begingroup$
            I would argue that $|B_{n}-B_{m}|=|sumlimits_{k=1}^{n} x_{k}-sumlimits_{k=1}^{m} x_{k}|=|sumlimits_{k=m+1}^{n} x_{k}|=sumlimits_{k=m+1}^{n} |x_{k}|=A_{n}-A_{m}$. Right?
            $endgroup$
            – Mathbeginner
            Mar 10 at 22:22












          • $begingroup$
            $|sum x_k|$ is not the same thing as $sum |x_k|$. The basic fact about $|.|$ is the triangle inequality $|sum x_k| leq sum |x_k|$. If you never heard about that, it is normal you are confused with absolute convergence questions ;)
            $endgroup$
            – TheSilverDoe
            Mar 10 at 22:24










          • $begingroup$
            You're right, I should have written $|B_{n}-B_{m}| leq A_{n}-A_{m}$, but then I could use the same road I did, right?
            $endgroup$
            – Mathbeginner
            Mar 10 at 22:26
















          1












          $begingroup$

          You should use Cauchy criterion.



          For $varepsilon > 0$, because $(A_n)$ converges there exists $N$ such that for $p > q geq N$,
          $$|A_p - A_q| leq varepsilon, text{ i.e. } sum_{k=q+1}^p |x_k| leq varepsilon$$



          So for $p>q geq N$,
          $$|B_p - B_q| = left| sum_{k=q+1}^p x_k right| leq sum_{k=q+1}^p |x_k| leq varepsilon$$



          This proves that $(B_n)$ is a Cauchy sequence, and if your space is complete, it converges.






          share|cite|improve this answer











          $endgroup$













          • $begingroup$
            I see what you mean and how this would work. But does that also mean my approach is wrong?
            $endgroup$
            – Mathbeginner
            Mar 10 at 22:16










          • $begingroup$
            You say that $|B_n - B_m|=A_n-A_m$, but this has no reason to be true in general...
            $endgroup$
            – TheSilverDoe
            Mar 10 at 22:19










          • $begingroup$
            I would argue that $|B_{n}-B_{m}|=|sumlimits_{k=1}^{n} x_{k}-sumlimits_{k=1}^{m} x_{k}|=|sumlimits_{k=m+1}^{n} x_{k}|=sumlimits_{k=m+1}^{n} |x_{k}|=A_{n}-A_{m}$. Right?
            $endgroup$
            – Mathbeginner
            Mar 10 at 22:22












          • $begingroup$
            $|sum x_k|$ is not the same thing as $sum |x_k|$. The basic fact about $|.|$ is the triangle inequality $|sum x_k| leq sum |x_k|$. If you never heard about that, it is normal you are confused with absolute convergence questions ;)
            $endgroup$
            – TheSilverDoe
            Mar 10 at 22:24










          • $begingroup$
            You're right, I should have written $|B_{n}-B_{m}| leq A_{n}-A_{m}$, but then I could use the same road I did, right?
            $endgroup$
            – Mathbeginner
            Mar 10 at 22:26














          1












          1








          1





          $begingroup$

          You should use Cauchy criterion.



          For $varepsilon > 0$, because $(A_n)$ converges there exists $N$ such that for $p > q geq N$,
          $$|A_p - A_q| leq varepsilon, text{ i.e. } sum_{k=q+1}^p |x_k| leq varepsilon$$



          So for $p>q geq N$,
          $$|B_p - B_q| = left| sum_{k=q+1}^p x_k right| leq sum_{k=q+1}^p |x_k| leq varepsilon$$



          This proves that $(B_n)$ is a Cauchy sequence, and if your space is complete, it converges.






          share|cite|improve this answer











          $endgroup$



          You should use Cauchy criterion.



          For $varepsilon > 0$, because $(A_n)$ converges there exists $N$ such that for $p > q geq N$,
          $$|A_p - A_q| leq varepsilon, text{ i.e. } sum_{k=q+1}^p |x_k| leq varepsilon$$



          So for $p>q geq N$,
          $$|B_p - B_q| = left| sum_{k=q+1}^p x_k right| leq sum_{k=q+1}^p |x_k| leq varepsilon$$



          This proves that $(B_n)$ is a Cauchy sequence, and if your space is complete, it converges.







          share|cite|improve this answer














          share|cite|improve this answer



          share|cite|improve this answer








          edited Mar 10 at 22:27

























          answered Mar 10 at 22:15









          TheSilverDoeTheSilverDoe

          3,866112




          3,866112












          • $begingroup$
            I see what you mean and how this would work. But does that also mean my approach is wrong?
            $endgroup$
            – Mathbeginner
            Mar 10 at 22:16










          • $begingroup$
            You say that $|B_n - B_m|=A_n-A_m$, but this has no reason to be true in general...
            $endgroup$
            – TheSilverDoe
            Mar 10 at 22:19










          • $begingroup$
            I would argue that $|B_{n}-B_{m}|=|sumlimits_{k=1}^{n} x_{k}-sumlimits_{k=1}^{m} x_{k}|=|sumlimits_{k=m+1}^{n} x_{k}|=sumlimits_{k=m+1}^{n} |x_{k}|=A_{n}-A_{m}$. Right?
            $endgroup$
            – Mathbeginner
            Mar 10 at 22:22












          • $begingroup$
            $|sum x_k|$ is not the same thing as $sum |x_k|$. The basic fact about $|.|$ is the triangle inequality $|sum x_k| leq sum |x_k|$. If you never heard about that, it is normal you are confused with absolute convergence questions ;)
            $endgroup$
            – TheSilverDoe
            Mar 10 at 22:24










          • $begingroup$
            You're right, I should have written $|B_{n}-B_{m}| leq A_{n}-A_{m}$, but then I could use the same road I did, right?
            $endgroup$
            – Mathbeginner
            Mar 10 at 22:26


















          • $begingroup$
            I see what you mean and how this would work. But does that also mean my approach is wrong?
            $endgroup$
            – Mathbeginner
            Mar 10 at 22:16










          • $begingroup$
            You say that $|B_n - B_m|=A_n-A_m$, but this has no reason to be true in general...
            $endgroup$
            – TheSilverDoe
            Mar 10 at 22:19










          • $begingroup$
            I would argue that $|B_{n}-B_{m}|=|sumlimits_{k=1}^{n} x_{k}-sumlimits_{k=1}^{m} x_{k}|=|sumlimits_{k=m+1}^{n} x_{k}|=sumlimits_{k=m+1}^{n} |x_{k}|=A_{n}-A_{m}$. Right?
            $endgroup$
            – Mathbeginner
            Mar 10 at 22:22












          • $begingroup$
            $|sum x_k|$ is not the same thing as $sum |x_k|$. The basic fact about $|.|$ is the triangle inequality $|sum x_k| leq sum |x_k|$. If you never heard about that, it is normal you are confused with absolute convergence questions ;)
            $endgroup$
            – TheSilverDoe
            Mar 10 at 22:24










          • $begingroup$
            You're right, I should have written $|B_{n}-B_{m}| leq A_{n}-A_{m}$, but then I could use the same road I did, right?
            $endgroup$
            – Mathbeginner
            Mar 10 at 22:26
















          $begingroup$
          I see what you mean and how this would work. But does that also mean my approach is wrong?
          $endgroup$
          – Mathbeginner
          Mar 10 at 22:16




          $begingroup$
          I see what you mean and how this would work. But does that also mean my approach is wrong?
          $endgroup$
          – Mathbeginner
          Mar 10 at 22:16












          $begingroup$
          You say that $|B_n - B_m|=A_n-A_m$, but this has no reason to be true in general...
          $endgroup$
          – TheSilverDoe
          Mar 10 at 22:19




          $begingroup$
          You say that $|B_n - B_m|=A_n-A_m$, but this has no reason to be true in general...
          $endgroup$
          – TheSilverDoe
          Mar 10 at 22:19












          $begingroup$
          I would argue that $|B_{n}-B_{m}|=|sumlimits_{k=1}^{n} x_{k}-sumlimits_{k=1}^{m} x_{k}|=|sumlimits_{k=m+1}^{n} x_{k}|=sumlimits_{k=m+1}^{n} |x_{k}|=A_{n}-A_{m}$. Right?
          $endgroup$
          – Mathbeginner
          Mar 10 at 22:22






          $begingroup$
          I would argue that $|B_{n}-B_{m}|=|sumlimits_{k=1}^{n} x_{k}-sumlimits_{k=1}^{m} x_{k}|=|sumlimits_{k=m+1}^{n} x_{k}|=sumlimits_{k=m+1}^{n} |x_{k}|=A_{n}-A_{m}$. Right?
          $endgroup$
          – Mathbeginner
          Mar 10 at 22:22














          $begingroup$
          $|sum x_k|$ is not the same thing as $sum |x_k|$. The basic fact about $|.|$ is the triangle inequality $|sum x_k| leq sum |x_k|$. If you never heard about that, it is normal you are confused with absolute convergence questions ;)
          $endgroup$
          – TheSilverDoe
          Mar 10 at 22:24




          $begingroup$
          $|sum x_k|$ is not the same thing as $sum |x_k|$. The basic fact about $|.|$ is the triangle inequality $|sum x_k| leq sum |x_k|$. If you never heard about that, it is normal you are confused with absolute convergence questions ;)
          $endgroup$
          – TheSilverDoe
          Mar 10 at 22:24












          $begingroup$
          You're right, I should have written $|B_{n}-B_{m}| leq A_{n}-A_{m}$, but then I could use the same road I did, right?
          $endgroup$
          – Mathbeginner
          Mar 10 at 22:26




          $begingroup$
          You're right, I should have written $|B_{n}-B_{m}| leq A_{n}-A_{m}$, but then I could use the same road I did, right?
          $endgroup$
          – Mathbeginner
          Mar 10 at 22:26


















          draft saved

          draft discarded




















































          Thanks for contributing an answer to Mathematics Stack Exchange!


          • Please be sure to answer the question. Provide details and share your research!

          But avoid



          • Asking for help, clarification, or responding to other answers.

          • Making statements based on opinion; back them up with references or personal experience.


          Use MathJax to format equations. MathJax reference.


          To learn more, see our tips on writing great answers.




          draft saved


          draft discarded














          StackExchange.ready(
          function () {
          StackExchange.openid.initPostLogin('.new-post-login', 'https%3a%2f%2fmath.stackexchange.com%2fquestions%2f3142980%2fproof-of-convergence-for-non-absolute-sequence%23new-answer', 'question_page');
          }
          );

          Post as a guest















          Required, but never shown





















































          Required, but never shown














          Required, but never shown












          Required, but never shown







          Required, but never shown

































          Required, but never shown














          Required, but never shown












          Required, but never shown







          Required, but never shown







          Popular posts from this blog

          Magento 2 - Add success message with knockout Planned maintenance scheduled April 23, 2019 at 23:30 UTC (7:30pm US/Eastern) Announcing the arrival of Valued Associate #679: Cesar Manara Unicorn Meta Zoo #1: Why another podcast?Success / Error message on ajax request$.widget is not a function when loading a homepage after add custom jQuery on custom themeHow can bind jQuery to current document in Magento 2 When template load by ajaxRedirect page using plugin in Magento 2Magento 2 - Update quantity and totals of cart page without page reload?Magento 2: Quote data not loaded on knockout checkoutMagento 2 : I need to change add to cart success message after adding product into cart through pluginMagento 2.2.5 How to add additional products to cart from new checkout step?Magento 2 Add error/success message with knockoutCan't validate Post Code on checkout page

          Fil:Tokke komm.svg

          Where did Arya get these scars? Unicorn Meta Zoo #1: Why another podcast? Announcing the arrival of Valued Associate #679: Cesar Manara Favourite questions and answers from the 1st quarter of 2019Why did Arya refuse to end it?Has the pronunciation of Arya Stark's name changed?Has Arya forgiven people?Why did Arya Stark lose her vision?Why can Arya still use the faces?Has the Narrow Sea become narrower?Does Arya Stark know how to make poisons outside of the House of Black and White?Why did Nymeria leave Arya?Why did Arya not kill the Lannister soldiers she encountered in the Riverlands?What is the current canonical age of Sansa, Bran and Arya Stark?